What is Rod: Definition and 1000 Discussions

Röd (Swedish for Red) is the eighth studio album by Swedish alternative rock band Kent. It was released as digital download exclusively through the band's website on 5 November 2009 and physically on 6 November 2009. The first single from the album, "Töntarna", was released as digital download on 5 October 2009.
Röd is available in a standard edition and a deluxe edition box. The deluxe edition box version features the 11-track CD, a USB flash drive with high quality MP3 files as well as AIFF files, three 10" records which between them contain the whole album, and a 118-page book containing lyrics, abstract pictures and photographs. Due to distribution difficulties the deluxe edition was delayed until 11 November 2009.

View More On Wikipedia.org
  1. S

    EMF induced in rotating rod inside uniform magnetic field

    Homework Statement A 40 cm rod is rotated about its centre inside a region of uniform magnetic field of 6.4 T. Given that the speed of rotation is 15 rad/s, find potential difference between the centre and either end of the rod Homework Equations emf = - ΔΦ / Δt ω = 2π / T The Attempt at a...
  2. S

    Sliding Rod Problem: Find Velocity of Particle A at H=30°"

    Homework Statement Two particles A and B are connected by a rigid rod AB which slides along two mutually perpendicular rails as shown. H is the angle between point B and the horizontal rod. At an instant, when H=30°, the velocity of B is 10m/s towards left, and velocity of A is upwards. Find...
  3. K

    Heat diffusion into an infinite rod

    I have recently been curious about heat diffusion. If there is space in one dimension with any kind of temperature dispersed throughout, then the heat equation states that the derivative of the temperature with respect to time at any point equals some constant (k) multiplied by the second...
  4. G

    Stress, Strain, and Sound in a Projectile Steel Rod

    Homework Statement .[/B] For a certain type of steel, stress is always proportional to strain with Young's modulus 20 x 10^10 N/m^2. The steel has density 7.86 x 10^3 kg/m^3. A rod 80.0 cm long, made of this steel, is fired at 12.0 m/s straight at a very hard wall. a) The speed of a...
  5. iVenky

    Surface charge on a conductor due to a charged rod 'q'

    Homework Statement A charged rod of charge 'q' is at a distance 'd' from a perfect conductor as shown below. What's the total surface charge on the conductor? 2. Homework Equations I tried to solve this without equations. The Attempt at a Solution [/B] Basically, as long as there is E field...
  6. A

    Moment of inertia of a thin uniform rod

    I was thinking that if a uniform rod of mass M and length L remains static ,then it's centre of mass will be at L/2 from one end (e.g total mass assumed to be concentrated at L/2 ) But if this rod is moving with uniform angular velocity ω about an axis passing through it's one end and...
  7. person123

    Thermal Expansion of a Wire Connected To a Rod

    Homework Statement Homework Equations ΔL=LαΔT σ=EΔL/L The Attempt at a Solution For part a, I used the coefficient of linear expansion for copper and the change in temperature to find the change in length (0.068%). I thought part b had the same answer. The two are attached, and I would...
  8. P

    Finding the mass of a ball and a rod using torque

    Homework Statement There is a diagram of a rod with its center of mass labeled and a ball hanging down from it, attached by a string. Two torques are applied on either side. You are asked to find the mass of the ball and the mass of the rod. Homework Equations Torque = Force * Lever Arm *...
  9. J

    Rod on rails in a magnetic field

    Homework Statement A rod with mass of m runs on rails in a magnetic field. The rails are connected with a wire at x = 0 so that the rod and the rails together with this wire form a closed circuit. The distance between the rails is L, and the resistance in the rod is R. We assume that the...
  10. Krushnaraj Pandya

    Inelastic collision of ball and rod - rotation problem

    Homework Statement A uniform rod of mass M and length l is hinged at the center. a particle of mass m and speed u sticks after hitting the end of the rod. find the angular velocity of the rod just after collision Homework Equations Energy conservation-0.5mu^2=0.5(m+M)v^2 Angular momentum...
  11. J

    Small oscillation frequency of rod and disk pendulum

    Homework Statement Consider a rod of length ##L## and mass ##M## attached on one end to the ceiling and on the other end to the edge of a disk of radius ##r## and mass ##m##. This system is slightly moved away from the vertical and let go. Let ##\theta## be the angle the pendulum makes with the...
  12. isukatphysics69

    Find electric field a distance P from a charged rod

    Homework Statement The figure below shows a thin, vertical rod of length L with total charge Q. The indicated point P is a horizontal distance x from the one end of the rod. What is the electric field at point P. Express your answer in component notation in the two blanks below. L = 5.0 cm, Q...
  13. Navin

    Two blocks on a plane connected with a Rod

    Homework Statement Two blocks m1 and m2 are connected by a weightless rod are on a inclined plane of ∅=37° and μ =0.25 .What is the common acceleration and Tension of rod if m1 is 4 kg and m2 is 2 kg such that the 4kg block is the upper block. OPTIONS A =4m/2 T=0 B =2m/2 T=5N C =10m/2 T=10N D...
  14. A

    Calculate the relative angular velocity

    Homework Statement AB is a rod of length 10 m that is leaning against the wall. Given variables are shown in the diagram. Find angular velocity of A wrt B. https://imgur.com/a/8bEdYhN I have a doubt in one step that I will highlight in "The attempt at a solution" part. Homework Equations...
  15. H

    What makes a connecting rod turn a crank one way?

    This may be the most awkward unexpected question ever concerning the reasoning for which I ask it. I begin with explaining the reason because I think you should know why I ask this question to begin with so you can understand what underlying basic principle of this mechanic I want to...
  16. J

    Calculating Max Pull for M30x2mm Threads on Chrome Rod

    Hi, I'm machining threads on to the ends of chrome rod for use in hydraulic rams. I have a ram which will see a maximum retracting load of 10 ton (98.1kN). For this I'm considering a thread of M30x2mm pitch. The chrome rod we use has a tensile strength of 550 n/mm2 with a yielding of 420 n/mm2...
  17. cookiemnstr510510

    Electric field of bent non conducting rod

    Homework Statement You are given a non conducting rod carrying uniformly distributed charge, -Q, that has been bent into a 120° circular arc of radius, R. The axis of symmetry of the arc lies along the x-axis and the origin is at the center of curvature of the arc. (a) in terms of Q and R...
  18. Amitayas Banerjee

    Equilibrium Conditions for a Rotating Rod with Two Point Masses

    1. A weightless rod carries towards of masses M and M. The roads Hinge Joint to vertical axis OO', which rotates with an angular velocity ω. Determine the angle φ formed by the rod and there vertical. The attempt at a solution If I am not wrong, the two ways to ensure equilibrium are...
  19. L

    A Momentum calculation of a rotating rod

    ##\ \ \ \ \ ##Calculate the 4 momentum of a rotating rod. We divide it into 4 parts. The part 1 is the work of predecessors. ##\ \ \ \ \ ##In Special relativity, the motion of rod AB (which is an object in non inertial motion) can be described in an inertial reference frame and the motion of rod...
  20. S

    Rod w/ Pivot and Attached Mass Pendulum

    Homework Statement A solid rod of length L and mass M has a pivot through its center and is originally horizontal. Another mass 2M is then attached firmly to one end of the rod, and released. What is the maximum speed of the mass 2M attained thereafter? (Cornell 2009) Homework Equations Not...
  21. V

    Mass needed to balance the magnetic force on upper rod

    Homework Statement Two straight rods 60 cm long and 2.0 mm apart in a current balance carry currents of 18 A each in opposite directions. What mass must be placed on the upper rod to balance the magnetic force of repulsion? ## \mu_0 = 4 \pi * 10^-7 ~\frac {T * M} {A}## ## g = 9.81 ~m/s ##...
  22. Krushnaraj Pandya

    Angular momentum about ICOR of a rod

    Homework Statement A rod (mass M, length L) is placed vertically on a smooth horizontal surface. Rod is released and after some time velocity of COM is v downwards and at this moment rod makes 60 degrees with horizontal. Find angular momentum of rod about Instantaneous center of rotation...
  23. Krushnaraj Pandya

    How is net torque zero about hinge of rod in ball rod system

    Homework Statement There are 2 situations- 1)a rod of length L is lying on a smooth horizontal surface with its one end hinged to the ground, a ball traveling with a velocity v hits its other end and comes to rest, How is the net external torque about hinge point O zero? 2) in the second case...
  24. Krushnaraj Pandya

    Angular momentum of a rod about hinge

    Homework Statement A uniform rod (M, L) is rotated about a point L/3 from its left end. Angular momentum about O Homework Equations 1) L=I(cm)w for purely rotating body 2) L(orbital)= M*v(cm)*perpendicular distance(r) 3) L(spin)= I*w The Attempt at a Solution I got the correct answer in two...
  25. S

    Dielectric Rod Antenna: Can It Radiate?

    I have a fundamental question about the antennas made by using dielectric materials that I have not found a satisfied answer online. According to Wikipedia, antennas are described as, "In radio, an antenna is the interface between radio waves propagating through space and electric currents...
  26. A

    Charge on a rod of infinite length

    1. Charge is distributed through an infinitely long cylinder of radius R in such a way that the charge density is proportional to the distance from the central axis: ß = A r, where A is a constant and ß is the density. (a) Calculate the total charge contained in a segment of the cylinder of...
  27. A

    Feynman Exercises 19-1: Metal rod framework being pulled in while spinning

    Homework Statement Homework Equations The Attempt at a Solution The moment of inertia before collapse is for each rod: BEFORE COLLAPSE:[/B] Ib = ∫(L2 + x2) dm = m/L ∫(L2 + x2) dx = 4/3 * m * L2 We have 8 of these plus the inertia of the mechanism, giving a total I, It = 8 * Ib + Ik =...
  28. B

    How does the incline of the rod depend on mass and force F?

    Homework Statement We have a rod with length L and with the left end the rod is fixed to the ceiling in such a way it can swivel. As we can see from the picture (attached file) on right end two forces are acting on the rod: F and mg. The question is: How does the incline of the rod depend on...
  29. scoutdjp2012

    Conducting rod creates a complete circuit

    Homework Statement A conducting rod makes contact with rails to complete a circuit. If the rails are 50 cm apart ina uniform magnetic field B = 0.38 [T] directed out of the paper. The total resistance of the circuit is R = 9 [ohms] and is constant. 1) what is the magnitude and direction of EMF...
  30. B

    Direction of The Force of a Wall Exerted on a Rod

    Homework Statement A rod is attached to a wall in such a way it can swivel. In this case: In which direction does the force (of the wall on the rod) point to? Here are two examples (see under attached files), but the the direction of the force is different. Why? Is maybe one of the pictures...
  31. Zubair Ahmad

    Momentum Conservation: Ball Hitting Pivoted Rod

    If a ball hits a rod at the top which is pivoted at bottom end then is linear momentum conserved?
  32. S

    Moving a conducting rod in a magnetic field

    Homework Statement A small conducting rod of length l moves with a uniform velocity v in a uniform magnetic field B. Explain how the end Y becomes positively charged. Homework EquationsThe Attempt at a Solution I know the concept, a force will act on the conductor which will push the...
  33. Hamza Abbasi

    Strain produced in a rod after expansion

    Homework Statement A rod of length ##L_o## is kept on a friction-less surface. The coefficient of linear expansion for the material of the rod is ##\alpha##. The the temperature of the rod is increased by ##\Delta T## the strain developed in the rod will be? Homework Equations $$ \Delta L=...
  34. B

    A rod of length L and mass M has a nonuniform mass distribut

    Homework Statement A rod of length L and mass M has a nonuniform mass distribution. The linear mass density (mass per length) is λ=cx2, where x is measured from the center of the rod and c is a constant. Find the expression for c.Find the expression for the moment of inertia of the rod for...
  35. J

    Bent rod rotating in a magnetic field

    Homework Statement Homework EquationsThe Attempt at a Solution I am considering the two rods separately .Considering QP alone EMF induced will be (1/2)Bωl2 with Q at higher potential . Considering QR alone EMF induced will be (1/2)Bωl2 with Q at higher potential . This gives zero potential...
  36. Alcatrez

    Uniform rod pivoting and block/mass energy problem

    A uniform rod of mass m and length l is pivoted at a point on the ground. The rod is initially in vertical position and touching a block of mass M which is at rest on a horizontal surface. The rod is given a push of negligible force and it starts rotating about that point. This causes the block...
  37. Hiero

    I A moving rod; two Lorentz boosts compared with one

    Consider a rod oriented along and moving along the x direction at a speed v in frame A. In it's turn, frame A is moving at speed u in the y direction relative to frame B. By my understanding of special relativity, the x component of the rod's velocity in frame B will be v/γu, and the y...
  38. Perodamh

    What is the diameter of the cylindrical rod

    Homework Statement A cylindrical rod of copper (E = 110GPa) having a yield strength of 240MPa is to be subjected to a load of 6660N. If the length of the rod is 380mm, what must be the diameter to allow an elongation of 0.5mm. Homework Equations E = stress/ strain stress = Force/Area ; Area =...
  39. M

    Hello, ferrite rod coupling inductor, I dont understand

    hi to everybody, can somebody tell me, ferrit rod coils, is a transformer? if yes how do you calculate? what is the formula how you select the coupling inductor L2 or L3... because like i read the maing inductor must connect to capacitor(for frequency to find min and max i understand) but...
  40. M

    Hi, I with a ferrite rod antenna please

    hello to everybody, can some help to make a ferrit rod antenna and to explain me thnx is for am radio, trf regenerative i have a ferrit rod 90mm diameter, and i have 0.5mm wire magnetic. this is the schematic , i make also the am receiver, but i don't know the ferrit rod(is problem i try to...
  41. T

    Pulling/pushing an elastic rod at high speeds

    Consider an elastic rod lying on a table. If one end of the rod is pulled/pushed along the length of the rod with speed v, the other end will not immediately start moving, because any disturbance takes time to propagate along the rod. To be precise, the other end will move after a time t=L/c...
  42. B

    Extension of a rod segment dx due to a passing longitudinal wave

    Let us look at short segment of a rod with its length dx. Due to longitudinal wave, left endpoint moves for s in the direction of x-axis and the right endpoint moves in the same direction for s+ds. Because I want to calculate the elastic energy of the wave motion, I need the extension of dx so...
  43. P

    B What are the physics behind fishing rods?

    Hi everyone, First post here. Been out of school for a while and quite rusty, but always had an interest in physics, and was pretty good back in the day. Can anyone help me understand the physics of fishing rods? How to look at a fishing rod in terms of physics and mechanics? (i.e. is it a...
  44. AntSC

    Moments - Support forces acting on a rod

    Homework Statement This is more of a question of applying the correct principles to a range of problems involving a horizontal rod hinged to a wall that is then supported by another rod. The support rod is also attached to the wall from above or below and then attached to the horizontal rod, so...
  45. Brilli

    An equilibrium problem -- Spinning a hinged rod and a ball

    Homework Statement This is a practice olympiad problem A light rod with length l is hinged in such a way that the hinge folds in one plane only. The hinge is spun with angular speed ω around a vertical axis. A small ball is fixed to the other end of the rod. (a) Find the angular speeds for...
  46. O

    Inserting a steel rod into a current-carrying copper pipe

    << Mentor Note -- thread moved from the technical forums, so no Homework Template is shown >> The situation is as follows: Current I flows along the axis of the copper pipe. From Ampere's law, the magnetic field inside the pipe is zero and is equal to μ0*I/(2π*r) outside the pipe. Now, an...
  47. F

    I Gauss’ law and an infinite rod

    To find the electric field from an infinitely long charged rod you can use gauss’s law with a cylinder as your Gaussian surface. I don’t quite understand by this works. Wouldn’t the electric field given by the equation only be the electric field cause by the charge within the cylinder? And if...
  48. F

    Oscillation of a Rod: Finding Frequency

    Homework Statement As in picture b) a uniform rod is displaced by a small angle a, the question is to find its frequency. (small angle approximations are allowed) Homework Equations Mtotal*a c.o.m=FextThe Attempt at a Solution I figure the only relevant external force is the komponent of...
  49. D

    Fall down of an long elastically vertically placed rod

    Let`s observe fall down of an long elastic vertically placed rod which has two point A and B. Point A is being placed on the ground and not moving, Point B is being free. We observe that the free ending is bending backwards and gets an arc shaped form. In your option, is it an optical...
  50. klm_spitifre

    Electrical field near infinite, charged rod

    Homework Statement Calculate the electric field at point ##P## if the distance from the center of an infinitely long, charged rod to point ##P## is ##a = 0.6m##; the charge density equals ##\lambda = -CX^2##, ##C=10^{-3}C/m^3##. Show all steps in finding the equation of the field, then find the...
Back
Top